Econ study set

अब Quizwiz के साथ अपने होमवर्क और परीक्षाओं को एस करें!

Which of the following statements regarding a Production Possibilities Frontier is true? A. "Output combinations beyond the frontier are unattainable (i.e., not feasible)." B. "The attainable (i.e., feasible) points characterized by productive efficiency are below the curve." C. "The frontier will be positively sloped revealing that workers can collectively produce more total output when they 'specialize in production' (and each focus their production on the good for which they possess an Absolute Advantage)." D. "The frontier will become flatter (i.e., a slope closer to zero in absolute terms) as one moves 'down' the curve, since when expanding production of the good on the x-axis, a society should always use the available productive resource with the highest valued Opportunity Cost for producing the good."

A. "Output combinations beyond the frontier are unattainable (i.e., not feasible)." Explanation?

Which of the following two statements is a "Positive Statement"? A. "The American rock band Sponge will be performing at Madlife Stage and Studios in Woodstock, GA on Wednesday, November 1, 2023." B. "The song 'Plowed' by Sponge is the best song released in the 1990s." C. Both "A" and "B" are positive statements. D. Neither "A" nor "B" is a positive statement.

A. "The American rock band Sponge will be performing at Madlife Stage and Studios in Woodstock, GA on Wednesday, November 1, 2023." Explanation?

Wade owns a lawn care service. Last month he had Revenue of $6,500, Explicit Costs of $4,000, and Implicit Costs of $3,500. It follows that he earned an Economic Profit of A. - $1,000 (negative $1,000). B. $1,000. C. $2,500. D. $3,000.

A. - $1,000 (negative $1,000). Explanation: Economic Profit = Revenue - (Explicit Costs + Implicit Costs) =$6,500−($4,000+$3,500) =$6,500−$7,500=−$1,000 Answer: A. -$1,000 (negative $1,000)

Consider a firm that produces output (denoted by Q) using three inputs (denoted by X, Y, and Z). When using X=6, Y=12, and Z=15, the firm produces Q=4,500 units of output. When using X=10, Y=20, and Z=25, the firm produces Q=6,750 units of output. Based upon these observations, the firm's production process appears to exhibit A. Diseconomies of Scale B. Economies of Scale C. Constant Scale Economies D. Vertical Scale Economies

A. Diseconomies of Scale Explanation: o determine the type of scale economies exhibited by the firm's production process, we can use the production function. The production function represents the relationship between inputs (X,Y,Z) and output (Q). We'll assume a Cobb-Douglas production function, which is commonly used in economic analysis: Q=A⋅Xa⋅Yb⋅Zc Where: Q is the quantity of output. X,Y,Z are the inputs. A is a constant. a,b,c are the elasticities of the inputs. Now, we can use the given information to estimate the values of a,b,candA. 1). First observation:- X=6,Y=12,Z=15,Q=4,500 4,500=A⋅6a⋅12b⋅15c 2). Second observation:- X=10,Y=20,Z=25,Q=6,750 6,750=A⋅10a⋅20b⋅25c Now, let;s divide the second equation by the first to eliminate A: 6,750/4,500=A⋅106a⋅20b⋅25c/A⋅6a⋅12b⋅15c Simplify this expression: 3/2=(10/6)a⋅(20/12)b⋅(25/15)c Now, simplify further: 3/2=(5/3)a⋅(5/3)b⋅(5/3)c Combine the terms with the same base: 3/2=(5/3)a+b+c Now, we can solve for a+b+c: a+b+c=log5/3(3/2) Step 2 Calculate this value using a calculator. The result is approximately -0.07. Now, we have a+b+c, but we need to determine the individual values of a,b,c. Since we have three variables and only one equation, we cannot uniquely determine the values. However, we can determine the type of scale economies based on the sum a+b+c: If(a+b+c>1), the production process exhibits increasing returns to scale. If(a+b+c=1), the production process exhibits constant returns to scale. If(a+b+c<1), the production process exhibits decreasing returns to scale. Since a+b+c=−0.07<1, the firm's production process appears to exhibit decreasing returns to scale.

For Question 5, consider a firm with Long Run Average Total Costs as illustrated below. (HW5) It appears as if the underlying production relationship of this firm exhibits A. Economies of Scale over all ranges of output. B. Constant Scale Economies over all ranges of output. C. Diseconomies of Scale over all ranges of output. D. Diseconomies of Scale at low levels of output, Constant Scale Economies at intermediate levels of output, and Economies of Scale at high levels of output.

A. Economies of Scale over all ranges of output. Explanation: Economies of scale are the cost advantages that enterprises obtain due to their scale of operation, and are typically measured by the amount of output produced per unit of time. A decrease in cost per unit of output enables an increase in scale.

Price controls were broadly imposed in an attempt to combat inflation by the federal government in the United States during the time when ______________ was President. A. Richard Nixon B. Ronald Reagan C. Donald Trump D. Joe Biden

A. Richard Nixon Explanation: In order to combat inflation, price controls, including price limits and wage controls, were widely implemented by the federal government in the United States during the end of the 1960s and the beginning of the 1970s under President Richard Nixon.

The ____________________ states that: people have unlimited wants but a limited amount of resources; therefore, people face tradeoffs, in that having more of one thing means getting by with less of something else. A. Scarcity Principle B. Cost-Benefit Principle C. Incentive Principle D. Principle of Increasing Opportunity Cost

A. Scarcity Principle Explanation: The principle of scarcity states that there are limited goods and services for unlimited wants. Thus, people need to make choices in order to satisfy the wants that are most important to them.

The ____________ refers to a period of time during which the amount used/hired of at least one input is fixed at a predetermined level. A. Short Run B. Intermediate Run C. Medium Run D. Long Run

A. Short Run Explanation: The definition refers to a time period when at least one input, like capital or land, is held constant while other inputs can be varied. This is a foundational concept in microeconomics, especially in the theory of production.

For Question 3 and 4, consider a market with Marginal Private Costs, Marginal Private Benefits, Marginal Social Costs, and Marginal Social Benefits as illustrated below. (HW9) It appears as if production/consumption of this good generates A. a positive externality, but no negative externality. B. a negative externality, but no positive externality. C. both a positive externality and a negative externality. D. neither a positive externality nor a negative externality. The efficient quantity of production/consumption is ________ units. A. 135 B. 165 C. 280 D. 365

A. a positive externality, but no negative externality. Explanation? D. 365 Explanation?

The "Law of Demand" states that A. all other factors fixed, a greater quantity of a good will be demanded at lower prices. B. demand for a good will increase when consumer income increases. C. an increase in demand is visually illustrated by a rightward shift of the demand curve. D. there cannot be excess demand at the market equilibrium.

A. all other factors fixed, a greater quantity of a good will be demanded at lower prices. Explanation: The law of demand states that there is an inverse relationship between the price of a good and the quantity demanded of that good. This means that as the price of a good increases, the quantity demanded of that good will decrease. All other things being equal, consumers will buy more of a good when the price is lower.

For Questions 4 and 5, consider the market illustrated below. The efficient quantity of trade is 2,620 units. (HW4 NO4) If a Price Floor of $7.20 is imposed in this market there would be a Deadweight Loss equal to A. areas e+f. B. areas e+f+g. C. area h. D. areas h+i. Compared to the free market equilibrium outcome, imposing a price floor of $7.20 would A. decrease Total Consumers' Surplus by areas b+e. B. would generate $7,599 of tax revenue for the government. C. increase Total Producers' Surplus by areas b+c+d. D. More than one of the above answers is correct (which covers, but is not limited to, the case of "all of the above answers are correct").

A. areas e+f Explanation: when the price floor is $7.20 this is greater than the equilibrium price then there is Dead weight loss created (DWL). DWL = area e+f When 7.20 meets the price/deman line, and before supply and demand meets, that dwl. so E + F A. decrease Total Consumers' Surplus by areas b+e. Explanation: compared to free trade and price floor condition at $7.20 Consumer surplus in free trade equilibrium= a + b + e Producer surplus in free trade equilibrium= d + c + f There is no dead weight loss Explanation: Area above the price line and under the demand curve considered as consumer surplus. And area below price line under the supply curve is producer surplus. Correct option A consumer surplus decrease by area (b + e) Government generated revenue equals to area of rectangle 'b' Government generated revenue= 2.20 × 1490 = $3278 (area of rectangle b). But in this case government will not generate any revenue because it is not a per unit tax. If the revenue not goes to government then this part will go to the producers. Before price floor producer surplus= d + c + f After price floor producer surplus= b + c + d

For Question 16, consider the linear Demand curve illustrated below (1ST HALF HW) If an increase in supply led to a decrease in price from $5.36 to $4.87, total consumer expenditures on this good would A. increase. B. decrease. C. remain constant (i.e., not change). D. None of the above answers are necessarily correct, since the graph does not convey enough information to determine how total consumer expenditures would change for this decrease in price.

A. increase. Explanation: Here is given one demand function. From the diagram we can see, the demand line intersects the horizontal axis at (84000,0) and intersects the vertical axis at (0,8) So, the equation of the graph will be P−P1/P2−P1 = Q−Q1/Q2−Q1 0r, P−0/8−0 = Q−84,000/0−84,000 or, −84,000P = 8Q−672,000 or, Q = 84,000 - 10,500p Given, the price is decreased from $5.36 to $4.87 When P = $5.36, Q=84,000−10,500×5.36=27,720 So, total expenditure of the consumer = P×Q=$5.36×27,720=$148,579.2 When P= $4.87 Q=84,000−10,500×4.87=32,865 So, the total expenditure of the consumer = P×Q=$4.87×32,865=$160,052.55 So, the total consumer expenditure is increased from $148,579.2 to $160,052.55 when the price is decreased from $5.36 to $4.87. The total consumer expenditure is increased from $148,579.2 to $160,052.55 when the price is decreased from $5.36 to $4.87. Correct answer will be OPTION A.

For Questions 6 and 7, consider the game below: (hw8) For this game A. neither Player X nor Player Y has a dominant strategy. B. Player X has a dominant strategy, but Player Y does not. C. Player Y has a dominant strategy, but Player X does not. D. both Player X and Player Y have a dominant strategy.

A. neither Player X nor Player Y has a dominant strategy. Explanation: When player Y chooses left, player X can choose top with a payoff of 12 or bottom with a payoff of 18. Thus, player X will choose bottom to maximize the payoff. • When player Y chooses right, player X can choose top with a payoff of 10 or bottom with a payoff of 8. Thus, player X will choose top to maximize the payoff. Explanation: Therefore, the strategy chosen by player X changes with a change in the strategy chosen by player Y. Hence, player X does not have a dominant strategy. • When player X chooses top, player Y can choose left with a payoff of 7 or right with a payoff of 3. Thus, player Y will choose left to maximize the payoff. • When player X chooses bottom, player Y can choose left with a payoff of 9 or right with a payoff of 15. Thus, player Y will choose right to maximize the payoff. Therefore, the strategy chosen by player Y changes with a change in the strategy chosen by player X.

The Short Run supply curve of a firm operating in a perfectly competitive market can be described as the portion A. of the Marginal Cost curve that that lies above the Average Variable Cost curve. B. of the Marginal Cost curve that that lies above the Average Total Cost curve. C. of the Average Fixed Cost curve that that lies below the Marginal Cost curve. D. of the Average Variable Cost curve that that lies above the Average Fixed Cost curve.

A. of the Marginal Cost curve that that lies above the Average Variable Cost curve. Explanation?

Consider a firm that maximizes profit in the Short Run in a perfectly competitive market by producing 16,000 units of output. This firm has Total Revenue of $40,000, Variable Costs of $32,000, and Fixed Costs of $5,000. It follows that the Producer's Surplus of this firm is ____________. A. $3,000 B. $8,000 C. $13,000 D. $35,000

B. $8,000 Explanation: The producer's surplus in the short run is the difference between total revenue (TR) and variable costs (VC). Producer′Surplus=TR−VC Given that TR=$40,000, VC=$32,000, and Fixed Costs=$5,000, we can find the producer's surplus: Producer's Surplus=$40,000−$32,000=$8,000 So, the correct answer is B.$8,000.

Suppose that a good which generates a significant externality is traded in a free, unregulated market. If the external effect is a positive externality, we would expect the free market to ____________________; if the external effect is a negative externality, we would expect the free market to ____________________. A. provide less than the efficient amount of the good, leading to a positive Deadweight Loss; provide more than the efficient amount of the good, leading to a positive Deadweight Loss. B. provide more than the efficient amount of the good, leading to a positive Deadweight Loss; provide less than the efficient amount of the good, leading to a positive Deadweight Loss. C. provide more than the efficient amount of the good, leading to a positive Deadweight Loss; provide less than the efficient amount of the good, leading to a negative Deadweight Loss. D. provide less than the efficient amount of the good,

A. provide less than the efficient amount of the good, leading to a positive Deadweight Loss; provide more than the efficient amount of the good, leading to a positive Deadweight Loss. Explanation?

Excess Capacity refers to A. the amount by which a firm's output is less than its Efficient Scale of Production. B. the amount of output a firm produces but is not able to sell. C. the level of output which minimizes Average Total Costs of Production. D. efforts by firms to coordinate their actions in an attempt to increase both total industry profit and individual profit of each firm.

A. the amount by which a firm's output is less than its Efficient Scale of Production. Explanation: In economics, excess capacity occurs when a firm is producing less than its efficient scale of production, which means it could produce more output with the same or lower costs. So, the correct answer is A. Excess Capacity refers to the amount by which a firm's output is less than its Efficient Scale of Production, where the efficient scale is the level of output that minimizes average total costs.

The Basic Circular Flow Diagram illustrates A. the broad interactions between Households and Firms, voluntarily exchanging money for goods/services and/or factors of production in markets. B. how the interaction between supply and demand results in a specific equilibrium price and quantity of trade. C. the limits on production that a society faces, by illustrating the maximum amount of one good that can be produced for every possible level of production of another good. D. the impact of the government imposing a per unit tax in a market.

A. the broad interactions between Households and Firms, voluntarily exchanging money for goods/services and/or factors of production in markets. Explanation: The Basic Circular Flow Diagram illustrates the broad interactions between Households and Firms, voluntarily exchanging money for goods/services and/or factors of production in markets. This diagram showcases the flow of money and goods/services between these two fundamental economic entities in an economy.

Answer Questions 3 and 4 based upon the information conveyed in the following graph (which illustrates two different levels of Marginal Benefits of an activity for a decision maker). Suppose throughout that the Marginal Costs of the activity are equal to a constant $6.95. (1ST HALF HW) Suppose that Marginal Benefits change from MB1 to MB2 (with Marginal Costs fixed at MC=$6.95). It follows that as a result of this change in Marginal Benefits, A. the optimal choice of Q would increase, consistent with the Incentive Principle. B. the optimal choice of Q would increase, contradicting the Incentive Principle. C. the optimal choice of Q would decrease, consistent with the Incentive Principle. D. the optimal choice of Q would decrease, contradicting the Incentive Principle.

A. the optimal choice of Q would increase, consistent with the Incentive Principle. Explanation: Marginal benefit changes from MB1 to MB2 Then MB2 interacts with MC at quantity = 2925 as shown in the diagram above. This interaction gives a higher q =2925. The more the marginal benefit rises, the more the production rises. This is known as incentive principle. So answer is A option.

The Incidence of a Tax refers to A. who bears the burden of the tax in terms of decreased welfare. B. who is legally responsible for "writing a check" to pay the tax to the government. C. how much tax revenue is generated by the tax. D. how easy it is to "discourage consumption" of a good by imposing a tax.

A. who bears the burden of the tax in terms of decreased welfare. Explanation: The "Incidence of tax" means the share of the "burden" of a tax between or among the buyers or sellers. Incidence of tax is often referred to as "burden of tax". Hence, it refers to who bears the burden of the tax in terms of decreased welfare.

For Questions 6 and 7, consider the game below: (hw8) Considering the Mixed Extension of this game (in which: Player X chooses Top with probability p and Bottom with probability 1-p, and Player Y chooses Left with probability q and Right with probability 1-q), there is a mixed strategy Nash Equilibrium for which A. 𝑝∗ = 3/5 and 𝑞∗ = 1/4 . B. 𝑝∗ = 12/17 and 𝑞∗ = 3/8 . C. 𝑝∗ = 5/6 and 𝑞∗ = 6/7 . D. None of the above answers are correct because, in fact, there is no mixed strategy Nash Equilibrium for this game.

A. 𝑝∗ = 3/5 and 𝑞∗ = 1/4 . Explanation: Player X chooses top with probability p and bottomwith probability (1−p). Therefore,When player Y chooses left, player Y receives a payoff of 7with probability p or a payoff of 9 with probability (1−p). Thus, player Y's expected payoff from left is E(L)=7p+9(1−p) E(L)=7p+9−9p E(L)=9−2p When player Y chooses right, player Y receives a payoff of3 with probability p or a payoff of 15 with probability (1-p).Thus, player Y's expected payoff from right is E(R)=3p+15(1−p) E(R)=3p+15−15p E(R)=15−12p At mixed-strategy Nash equilibrium, the expected payoffsare equal. E(L)=E(R) 9−2p=15−12p 12p−2p=15−9 10p=6 p=6/10 p=3/5 The value of p is 3/5 at mixed-strategy Nash equilibrium. Player Y chooses left with probability q and right with probability (1−q). Therefore, When player X chooses top, player X receives a payoff of 12 with probability q or a payoff of 10 with probability (1−q). Thus, player X's expected payoff from top is E(T)=12q+10×(1−q) E(T)=12q+10−10q E(T)=2q+10 When player X chooses bottom, player X receives a payoffof 18 with probability q or a payoff of 8 with probability (1-q). Thus, player X's expected payoff from bottom is E(B)=18a+8(1−a) E(B)=18a+8−8c E(B)=10q+8 At mixed-strategy Nash equilibrium, the expected payoffsare equal. E(T)=E(B) 2a+10=10a+8 10q−2q=10−8 8a=2 q=2/8 q=1/4 The value of q is 1/4 at mixed-strategy Nash equilibrium.

For Question 2, consider a monopolist facing demand as illustrated below and with marginal revenue and costs as illustrated below. (HW7) This monopolist would maximize profit by producing and selling ________ units. A. 85 B. 230 C. 285 D. 360

B. 230 Explanation: As we know that Monopolist profit would be maximum when,MR=MC From the given diagram,MR and MC curve intersect at Q=230. Hence this monopolist would maximize profit by selling 230 units.

Consider "Good X" and suppose: • price elasticity of demand for Good X = - 0.743 • price elasticity of supply for Good X = 1.268 • income elasticity of demand for Good X = 0.345 • cross price elasticity of demand for Good X with respect to the price of Good Y = 0.296 Based upon these values, which of the following statements is/are accurate? A. "Good X is a complement to Good Y." B. "Good X is a normal good." C. "The 'Law of Supply' is violated for Good X." D. More than one of the above answers is correct (which covers, but is not limited to, the case of "all of the above answers are correct")

B. "Good X is a normal good." Explanation: A. "Good X is a complement to Good Y." To determine if Good X is a complement to Good Y, we would need information about the sign of the cross-price elasticity of demand. A positive cross-price elasticity (0.296 in this case) suggests that Good X and Good Y are substitutes, not complements. So, this statement is not accurate. B. "Good X is a normal good." The income elasticity of demand for Good X is 0.345, which is positive. A positive income elasticity indicates that Good X is a normal good, meaning that as consumers' income increases, they buy more of Good X. So, this statement is accurate. C. "The 'Law of Supply' is violated for Good X." The price elasticity of supply for Good X is 1.268, which is greater than 1. A price elasticity of supply greater than 1 suggests that the supply of Good X is elastic, meaning that producers are responsive to price changes by increasing their supply. This does not violate the Law of Supply. So, this statement is not accurate. D. More than one of the above answers is correct (which covers, but is not limited to, the case of "all of the above answers are correct"). Only statement B, "Good X is a normal good," is accurate. Therefore, more than one of the above answers is not correct in this case.

Which of the following is one of the "Three Basic Economic Questions" that every society must address? A. "What is the best level to set for the minimum wage?" B. "What mix of goods and services should we create?" C. "How can we make the distribution of income and wealth 'more equal'?" D. "How can we reduce our 'carbon footprint'?"

B. "What mix of goods and services should we create?" Explanation: In general, there are 3 basic questions in economics that each society and economy has to answer. These three questions are often summarized as follows: What to produce? How to produce? For whom to produce?

For Questions 6, consider the market illustrated below. Demand for this good is given by the inverse demand function Pd (q) = 20 - 1/250 q . If price were to decrease from $12.40 to $11.80, Total Consumers' Surplus would increase by ________. A. $1,140 B. $1,185 C. $1,230 D. $8,405

B. $1,185 Explanation: Quantity demanded when the price is 12.40 = 1900 Quantity demanded when the price is 11.80 = 2050 Consumer surplus when the price is 12.40 = 1/2×1,900×(20−12.40)=7,220 Consumer surplus when the price is 11.80 = 1/2×2,050×(20−11.80)=8,405 Change in consumer surplus = 8405 - 1185 = $1185

Robbie is a ventriloquist. He's currently having a hard time making ends meet and needs to sell his ventriloquist puppet, Rex, to get money to pay his rent. His "seller's reservation price" for this item is $150. Jade's "buyer's reservation price" for this item is $450. Suppose Jade buys Rex from Robbie for $280. It follows that this transaction gives Jade a Consumer's Surplus of ________ and gives Robbie a Producer's Surplus of ________. A. $730; $430. B. $170; $130. C. $150; $150. D. $85; $65.

B. $170; $130. Explanation: Seller reservation price is the minimum price that you would accept for a product or service you are selling. Buyer reservation price- Its the maximum amount the buyer would be willing to pay for what you have to offer. Seller reservation price= $150 Buyer reservation price = $450 Thus jade had a consumer surplus of $450-280= $170 Thus producer surplus of Robbie = 280- 150= $130

Answer Questions 5 and 6 based upon the information conveyed in the following table, which states Erin's Total Benefits from consumption of "Good Y" each month (measured in dollars). (refer to HW1 NO.5 FOR PIC) Erin's Marginal Benefit of consuming the 4th unit of "Good Y" would be _______. A. $10 B. $40 C. $55 D. $600

B. $40 Explanation= MB (4th unit) = TB (4 units) - TB(3 units) 220-180 =40

For Questions 12 through 15, consider a market in which Demand is given by the inverse function 𝑷𝑫(𝒒) = 28−.004q and Supply is given by the inverse function 𝑷𝑺(𝒒) = 𝟐+.001 q At the market equilibrium outcome, Total Consumers' Surplus is equal to A. $37,440 B. $54,080 C. $67,600 D. $91,520

B. $54,080 Explanation: Consumer Surplus = (\frac{1}{2} \times \text{base} \times \text{height}) Consumer Surplus = (\frac{1}{2} \times (5200-0) \times (10-6) = 10400) Now, multiply by the number of units: Total Consumers' Surplus = (10,400×5,200=54,080,000) This corresponds to: B. $54,080

For questions 6 and 7, consider the following scenario. "Horizon" provides internet service to residential customers in metro Atlanta. "Horizon" offers the three plans described below, and each household has the option of self-selecting the plan which they individually prefer. (HW10) "Horizon" is engaging in A. 1st Degree Price Discrimination (or "Perfect Price Discrimination"). B. 2nd Degree Price Discrimination (or "Menu Pricing"). C. 3rd Degree Price Discrimination (or "Segmented Pricing"). D. 4th Degree Price Discrimination (or "Extortion Pricing"). Suppose that Cathy always wants to use 200 GB of data per month, Dan always wants to use 500 GB of data per month, and Eli always wants to use 900 GB of data per month. In order to consume these desired levels of service at lowest expenditure, Cathy should choose __________, Dan should choose __________, and Eli should choose _______

B. 2nd Degree Price Discrimination (or "Menu Pricing"). Explanation? B. Plan C; Plan B; Plan B. Explanation?

For Question 7 and 8, consider a situation in which a good can be produced/consumed at 8 different positive levels. Production/consumption provides benefits to "Producers/Consumers," but imposes an External Cost on "Third Parties." Relevant values of these benefits and costs are stated in the table below. (HW9) The efficient (i.e., socially best) level of production/consumption is ______. A. 0 B. 4 C. 5 D. 7 Suppose that property rights can be clearly and completely defined and that parties can negotiate with each other at low cost (and any agreement reached can be enforced). In this case, if the Producers/Consumers have the initial property right, we would expect that as a result of negotiation A. the level of output would be 4, and Third Parties would pay Producers/Consumers somewhere between $175 and $600 to reduce the amount of output down to this level. B. the level of output would be 4, and Pr

B. 4 Explanation? A. the level of output would be 4, and Third Parties would pay Producers/Consumers somewhere between $175 and $600 to reduce the amount of output down to this level. Explanation?

For Questions 12 through 15, consider a market in which Demand is given by the inverse function 𝑷𝑫(𝒒) = 28−.004q and Supply is given by the inverse function 𝑷𝑺(𝒒) = 𝟐+.001 q The government is going to impose a per unit tax in this market. Which of the following taxes would generate the most revenue for the government? A. A $5.20 per unit tax on sellers. B. A $13.00 per unit tax on buyers. C. A $20.80 per unit tax on buyers D. A $26.00 per unit tax on sellers.

B. A $13.00 per unit tax on buyers. Explanation?

For Questions 2 and 3, consider the following scenario. A monopolist has customers in two market segments, "A" and "B," which respectively have demand given by the linear inverse functions 𝑷 𝑫/𝑨(𝒒) = 𝟔𝟔𝟔𝟔 − 𝒒𝒒 and 𝑷 𝑫/𝑩(𝒒) = 𝟒5 − 𝒒. Costs of production for the firm are 𝑪(𝒒) = 𝟓q + 𝟏,𝟎𝟎𝟎 (i.e., constant marginal costs of $5 per unit produced and Fixed Costs of $1,000). If the seller must treat these two segments as one single market, he will set a price of 𝒑∗ = 𝟑0; customers in Segment A will buy 35 units and customers in Segment B will buy 15 units. This price results in profit of $250. If the monopolist is able to engage in 3rd Degree Price Discrimination, then (in comparison to the "single market" outcome described above), A. Consumers' Surplus in Segment A will be larger and Consumers' Surplus in Segment B will be sm

B. Consumers' Surplus in Segment A will be smaller and Consumers' Surplus in Segment B will be larger. Explanation? A. $300 Explanation?

Firms operating in a perfectly competitive market should expect that in the Long Run A. Economic Profit will be positive but Accounting Profit will be equal to zero. B. Economic Profit will be equal to zero but Accounting Profit will be positive. C. Economic Profit will be negative but Accounting Profit will be equal to zero. D. both Economic Profit and Accounting Profit will be equal to zero.

B. Economic Profit will be equal to zero but Accounting Profit will be positive. Explanation?

________ has the Absolute Advantage in producing watermelons; ________ has the Absolute Advantage in sugar. A. Harry; Tyler. B. Harry; Mitch. C. Mitch; Harry. D. Mitch; Mitch.

B. Harry; Mitch. Explanation: Harry has the absolute advantage in producing watermelons, as he can produce more watermelons per unit of time than Mitch or Tyler. This can be seen from the fact that the PPF is steeper between points "B" and "C" than it is between points "C" and "D" or "D" and "E". Mitch has the absolute advantage in producing sugar, as he can produce more sugar per unit of time than Harry or Tyler. This can be seen from the fact that the PPF is flatter between points "C" and "D" than it is between points "B" and "C" or "D" and "E".

The ____________ refers to a period of time during which the amount used/hired of every single input can be altered. A. Intermediate Run B. Long Run C. Short Run D. Variable Run

B. Long Run Explanation?

For Questions 4 and 5, consider the potential provision of a pure public good (that can be produced in five different positive levels: very low, low, medium, high, or very high). "Zero" production of the good results in both zero benefits and zero costs. The society in which this good is to be produced consists of three different types of people, with benefits for the different possible levels of provision as stated below. For example, if "Medium Provision" of the good is provided, each Type 1 person gets benefits of $95, each Type 2 person gets benefits of $120, and each Type 3 person gets benefits of $140. As stated in the first column, there are: 400,000 Type 1 people; 1,000,000 Type 2 people; and 600,000 Type 3 people (for a total of 2 million people in the society). Total Costs of providing the different levels of the good are stated in the bottom row. (HW10) The efficient level (i.e., Social Surplus m

B. Low Provision Explanation? C. Medium Provision Explanation?

____________________ is a market structure in which there are no substantial barriers to entry, but in which competing firms sell differentiated products. A. Perfect Competition B. Monopolistic Competition C. Oligopoly D. Monopoly

B. Monopolistic Competition Explanation: In monopolistic competition, many firms operate, able to enter and exit the market easily. They each offer unique products, differing in branding, quality, or features, giving them individuality in a crowded marketplace.

After the first four games of the season, L.A. Rams Quarterback Matthew Stafford was averaging 307.25 passing yards per game. In the week five game against the Eagles he threw for 222 passing yards. As a result, his average passing yards per game after five games was equal to ________ yards. A. 222 B. 264.625 C. 290.2 D. 529.25

C. 290.2 Explanation: Average for first 4 games = 307.25 passing yards per game Total passing yards for first 4 games = 4×307.25=1,229 Adding the yards from the fifth game: 1,229+222=1,451 New average after five games = 1,451/5=290.2 Answer: C. 290.2 yards

__________________ is a market structure in which there are no substantial barriers to entry, but in which competing firms sell differentiated products. A. Perfect Competition B. Monopolistic Competition C. Oligopoly D. Monopoly

B. Monopolistic Competition Explanation: In monopolistic competition, many firms operate, able to enter and exit the market easily. They each offer unique products, differing in branding, quality, or features, giving them individuality in a crowded marketplace. This product differentiation gives each firm a certain level of control over its pricing strategy, setting it apart from perfect competition where products are homogeneous.

Which of the following is NOT one of the "three basic elements of a game"? A. Players. B. Rules. C. Strategies. D. Payoffs.

B. Rules. Explanation: The three basic elements of a game are: Players: They are the decision maker in a game. The player can be an individual or a firm. Strategies: These are the course of action available to each player. Payoff: The third most basic element of a game which is missing in the given question is payoff. Payoffs are the outcome to each player at the end of the game. These are usually measured in level of utility obtained by the players.

A "medium of exchange" refers to A. an asset that is used as a means of holding wealth. B. an asset that is used as payment when purchasing a good or service. C. something that is used as a unit of measure for economic activity. D. a stable state for a system that will persist as long as outside factors do not change.

B. an asset that is used as payment when purchasing a good or service. Explanation: A medium of exchange, by definition, is the method used to buy and sell, be it barter or personal checks. paper bills and coins that are used to buy goods and services.

For Question 5 and 6, consider a market with Marginal Private Costs, Marginal Private Benefits, Marginal Social Costs, and Marginal Social Benefits as illustrated below. (HW9) At the unregulated, free-market outcome there would be a Deadweight Loss equal to ____________. A. areas a+b+c B. area d C. areas e+f D. areas d+e+f+g The efficient quantity of trade would result if the government imposed a Per Unit Tax of ________ on sellers. A. $10.80 B. $11.50 C. $18.00 D. $22.10

B. area d Explanation? A. $10.80 Explanation?

For Questions 7 and 8, consider the market illustrated below. (HW3 NO 7) At the market equilibrium outcome, Total Consumers' Surplus is equal to __________ and Total Producers' Surplus is equal to __________. A. zero; areas c+d+f+g. B. areas a+b+e; areas c+f. C. areas a+b+e; areas c+d+f+g. D. areas a+b+c+e+f; areas d+g. If 2,700 units were produced/consumed, there would be a Deadweight Loss equal to __________. A. areas a+b+c+d+e+f+g+i. B. areas d+g+h+i. C. area h. D. area i.

B. areas a+b+e; areas c+f. Explanation: Total consumer surplus = a + b +e Total producer surplus = c + f C. area h. Explanation: Deadweight loss would be equal to area h

Consider the market for Good W. In this market, Demand is given by the inverse function 𝑃𝑃𝐷𝐷(𝑞𝑞) = 22 − .0075𝑞𝑞 and Supply is given by the inverse function 𝑃𝑃𝑆𝑆(𝑞𝑞) = 4 + .0015𝑞𝑞. There would be __________ at a price of $8.75 A. excess demand B. excess supply C. neither excess demand nor excess supply D. both excess demand and excess supply

B. excess supply Explanation: Step 1: Find the Quantity Demanded Given the demand function PDD​(q)=22−0.0075q and the price of $8.75, we set PDD​(q)=8.75 and solve for q: 22−0.0075q=8.75 0.0075q=13.25 q≈1766.67 At a price of $8.75, the quantity demanded is approximately 1766.67. Step 2: Find the Quantity Supplied Given the supply function PSS​(q)=4+0.0015q and the price of $8.75, we set PSS​(q)=8.75 and solve for q: 4+0.0015q=8.75 0.0015q=4.75 q≈3166.67 At a price of $8.75, the quantity supplied is approximately 3166.67. Step 3: Analyze the Market Since the quantity demanded (1766.67) is less than the quantity supplied (3166.67) for $8.75, there will be an excess supply or surplus in the market.

For Questions 7 and 8, consider the market for Good X. The table below summaries the current market shares of different producers of this good, along with projected market shares two years from now if a proposed merger between two existing firms takes place. (HW7) 8. Based upon HHI, the FTC would consider this market to be ____________ in 2023 and projects it to be ____________ in 2025. A. not concentrated; moderately concentrated. B. moderately concentrated; moderately concentrated. C. moderately concentrated; highly concentrated. D. highly concentrated; highly concentrated.

B. moderately concentrated; moderately concentrated. Explanation: Answer to Question 8: For HHI, markets are typically considered: Not concentrated: HHI below 1,500 Moderately concentrated: HHI between 1,500 and 2,500 Highly concentrated: HHI above 2500 So, in 2023, the market is not concentrated, and in 2025, it becomes moderately concentrated. Thus, the answer is: B. moderately concentrated; moderately concentrated. Explanation: Question 8: In 2,023, the market is not too concentrated. But by 2,025, it's expected to become moderately concentrated, which suggests a move towards lesser competition, but not to a point of being highly concentrated.

For Questions 12 through 15, consider a market in which Demand is given by the inverse function 𝑷𝑫(𝒒) = 28−.004q and Supply is given by the inverse function 𝑷𝑺(𝒒) = 𝟐+.001 q Focusing on the 2,500th unit, the buyer of this unit has a __________________________ and the seller of this unit has a __________________________. A. "buyer's reservation price" of $10.00; "seller's reservation price" of $2.50. B. "buyer's reservation price" of $10.80; "seller's reservation price" of $2.70. C. "buyer's reservation price" of $18.00; "seller's reservation price" of $4.50. D. "buyer's reservation price" of $28.00; "seller's reservation price" of $2.00. There would be __________ at a price of $10. A. excess demand B. excess supply C. neither excess demand nor excess supply D. both excess demand and excess supply

C. "buyer's reservation price" of $18.00; "seller's reservation price" of $4.50. Explanation: At the 2,500th unit: Buyer's reservation price is P_D(2500) = 28−0.004(2,500)=18. Seller's reservation price is P_S(2500) = 2+0.001(2,500)=4.5. ------------------------------------- B. excess supply Explanation?

For Question 4, consider a firm operating in a perfectly competitive market in the Short Run with costs as illustrated below. (2ND HALF HW) If the per unit price of output in this market is $36.10, the firm maximizes profit by producing ________ units of output in the Short Run. A. 0 B. 1,160 C. 2,800 D. 5,600

C. 2,800 Explanation: When the price is 36.10, the firm will consider this price as given and it will use its MC schedule to determine the quantity. If AVC is less than the price, the firm will continue to produce and if it is higher than the price, the firm will not produce. Here MC is higher than AVC at 36.10 so this firm will produce 2800 units

For Questions 7 and 8, consider a firm with costs as partially summarized in the table below: (HW5) When hiring 2 workers to produce 500 units of output, Average Fixed Costs are equal to ________. A. $0.48 B. $0.60 C. $1.50 D. $1.98 When hiring 5 workers to produce 875 units of output, Marginal Costs are equal to ________. A. $1.20 B. $2.00 C. $6.00 D. $13.50

C. $1.50 Explanation: A. $1.20 Explanation: Solution: firstly we filled the table:- Workers Units of output Marginal Product of Labor 0 0 0 1 300 300 2 500 200 3 650 150 4 775 125 5 875 100 6 935 60 •Formula for calculating Marginal Product of Labor:- MPL = change in quantity ÷ change in labor Here, MPL is Marginal Product of Labor. Fixed cost Variable cost Total cost 750 0 750 750 120 870 750 240 990 750 360 1110 750 480 1230 750 600 1350 750 720 1470 •At the 1st worker 870 Total cost is given from this we calculate fixed cost because Total cost and Variable cost both of the cost is given so, we subtract variable cost of total cost. 870−120=750 Here, the 1st worker fixed cost is 750. Fixed cost is always fixed at the all level of output. • Total cost formula: Total Cost= Fixed cost + variable cost From this formula we calculated above total cost Now we calculate the Average fixed cost when hiring 2 workers to produce 500 units of output. Formula for Average fixed cost: Average fixed cost= Fixed cost ÷ Output When hiring 2 workers to produce 500 units of output, average fixed costs are equal to $1.50 . Putting the value in formula: AFC=750÷500 AFC=$1.50 Option C, is correct after calculating the Average fixed cost $1.50 . 8. When hiring 5 workers to produce 875 units of output, marginal costs are equal to $1.20 . Formula for Marginal cost: Marginal cost= change in Total Cost ÷ Change in Quantity Marginal cost= change in Total Cost ÷ Change in Quantity MC=(1,350−1,230)÷(875−775) MC=120÷100 MC=$1.20 Option A, is correct after calculating the answer $1.20 .

For Questions 3 and 4, consider a monopolist facing demand of 𝑫(𝒑) = 𝟔𝟎𝟎,𝟎𝟎𝟎/ 𝒑𝟐 . For this demand function, elasticity is 𝜺 = −𝟐 at every point along the demand curve. Suppose this monopolist has costs of 𝑪(𝒒) = 𝟓q + 𝟐𝟎,𝟎𝟎𝟎, for which marginal costs are MC(𝒒) = 𝟓. To maximize profit, this monopolist will charge a price of A. $2 and sell 150,000 units of output B. $5 and sell 24,000 units of output C. $10 and sell 6,000 units of output D. $20 and sell 1,500 units of output For this monopolist, A. Profit is positive, but Producer's Surplus is negative. B. Producer's Surplus is positive, but Profit is negative. C. both Producer's Surplus and Profit are positive. D. both Producer's Surplus and Profit are negative.

C. $10 and sell 6,000 units of output Explanation: We know ε = -2, and the demand curve is D(p)=600,000/p^2. So, we can calculate MR as follows: εMR=p×(1+1/ε) MR=p×(1+1/−2) MR=p×(1−1/2) MR=p×1/2 MR=p/2 Now, set MR equal to MC: p/2=5 Solving for p: p=10 To find the corresponding quantity, we can use the demand curve: D(p)=600,000/p^2 D(10)=600,000/10^2 D(10)=600,000/100 D(10)=6,000 So, the monopolist will charge $10 and sell 6,000 units of output. Therefore, the correct answer to question 3 is C: $10 and sell 6,000 units of output. ------------------------------ C. both Producer's Surplus and Profit are positive. Explanation?

For Questions 7 and 8, consider the market illustrated below. Supply in April is given by the curve SApril and supply in June is given by the curve SJune (demand is given by the curve D in both April and June). (hw2, no 7) The reservation price of the buyer of the 90th unit of this good is _______ Which of the following statements is correct? A. "Between April and June there was a decrease in supply, but no change in demand." B. "Between April and June the market equilibrium price and market equilibrium quantity both decreased." C. "At a price of $6.00 there would have been 'excess demand' in April but 'excess supply' in June." D. More than one of the above answers is correct (which covers, but is not limited to, the case of "all of the above answers are correct").

C. $10.90 Explanation: Reservation price of the buyer is the maximum amount that the buyer is willing to pay to buy the given commodity. (On the graph, 90th = 10.90) C. "At a price of $6.00 there would have been 'excess demand' in April but 'excess supply' in June." Explanation: Let us understand the correctness of each of the given statements a. This statement is not correct. Between April and June, there was a rightward shift in the supply curve which implies that there was an increase in supply without any change in demand. b. This statement is not correct. Between April and June, there was an increase in quantity from 165 units to 235 units and there was a decrease in price from $7.50 to $5.10 c. The statement is Correct. For $6, there would have been excess demand in April but excess supply in June. d. This statement is not correct. As out of the above only one statement is correct.

For Questions 6 and 7, consider the market with Demand given by the inverse demand function 𝑃𝐷(𝑞𝑞) = 45,000/𝑞 and Supply given by the inverse demand function 𝑃𝑆(𝑞) = 8 + 1/600 𝑞 . The market equilibrium outcome (which is efficient) is approximately 𝑝∗ ≈ 13.54 and 𝑞∗ ≈ 3,324. If the government imposed a per unit tax of $2 on sellers in this market, they would get tax revenue of A. $5,000. B. approximately $5,792. C. $6,000. D. approximately $6,648.

C. $6,000. Explanation: New supply function after imposition of tax: P−2=8+1/600q P=10+1/600q Equilibrium output: 45,000/q=10+1/600q Combine 1600 and q. 45,000/q=10+q/600 Find the LCD of the terms in the equation. 600q Multiply each term in 45,000/q=10+q/600 by 600q to eliminate the fractions. 27,000,000=6,000q+q2 Solve the equation. q=3,000,−9,000 q =3000. Tax revenue=q×tax=3,000×2=$6,000 Therefore, Right answer: (C)

For questions 7 and 8 consider a situation in which a seller of two different goods (X and Y) faces five different types of consumers (A, B, C, D, and E - as indicated in the first column below, there are 200 of each type of consumer in the market). The reservation prices of each type of consumer for each of the two goods are summarized in the table below. If presented with multiple purchase options, a consumer will buy according to the option which gives her the largest, non-negative consumer's surplus or not buy at all (if all options would give negative consumer's surplus). Suppose that the Marginal Cost of producing a unit of either good is a constant $10 per unit (the seller has no Fixed Costs of production) (HW11) If this seller engages in "Simple Monopoly Pricing," her maximum profit is ____________; if this seller engages in "Pure Bundling," her maximum profit is ____________. A. $19,000; $25 split

C. $64,000; $70,000. Explanation? A. The seller will earn profit of $79,000 from setting these prices. Explanation?

For Question 8, consider a firm in a perfectly competitive market in the Short Run with costs as illustrated below. (HW6) If the per unit price of output in this market is $9.00, the firm maximizes profit by producing ________ units of output in the Short Run. A. 0 B. 2,250 C. 2,750 D. 3,925

C. 2,750 Explanation: If the per unit price of output in this market is $ 9.00, the firm maximizes profit by producing units of output in the Short Run. It is given that in the short run the price of the output in this industry is $ 9.00 therefore the firm would be producing at the point where MC curve is equal to $ 9.00. From the graph we can see the MC = $ 9.00, at an output level of 2750 units. Therefore, the short-run output of the firm would be 2,750 units. Short-run profit is maximized at P = MC. While in the long run profit is maximized at the point where MC =ATC. If the per unit price of output in this market is $ 9.00, the firm maximizes profit by producing 2,750 units of output in the Short Run. Option c. 2,750

For questions 17 and 18 refer to the graph below, which illustrates Total Social Surplus in the market for Good Z as a function of the level of production/consumption of the good. (1st half HW) The efficient level of trade of Good Z is _____ units. A. 100 B. 215 C. 338 D. 503 If 432 units of Good Z were produced/consumed, there would be a Deadweight Loss of _______. A. $2,410 B. $4,700 C. $7,450 D. $9,740

C. 338 Explanation: The efficient level of trade is where the value of total surplus is maximized, i.e. the market doesn't incur any surplus loss. According to the graph, At Q = 338; Total Surplus is maximum at $7,110 Therefore, the efficient level of trade is option C) 338 units. Explanation: The reason it is not: A) At Q = 100; TS = -$5040 which is less than $7110 B) At Q = 215; TS = $0 which is less than $7110 D) At Q = 503; TS = $0 which is less than $7110 -------------------------- A. $2,410 Explanation: Deadweight Loss refers to the loss of total surplus owing to the disequilibrium in the market. Thus, Deadweight Loss=Total SurplusEfficient Q−Total SurplusGiven Q We calculated that, Total Surplus (Efficient Q) = $7,110 Provided, Q = 432; i.e. total surplus = $4700 Hence, Deadweight Loss=$7,110−$4,700=$2,410 Explanation: The correct answer is option A) $2410

For Questions 7 and 8, consider the following scenario. Oliver and Anthony spend their workdays producing fudge rounds and pork. The table below provides a summary of the number of units of each good that each worker could produce in a full week (notice that Oliver's output of fudge rounds is generically given by "F"). Answer the following questions based upon these values. (HW1 NO7) Anthony's Opportunity Cost for producing a unit of pork is ________ units of fudge rounds. A. . 25 = 75 ÷ 300 B. 3 = 75 ÷ 25 C. 4 = 300 ÷ 75 D. 225 = 300 − 75 Oliver possess a Comparative Advantage in the production of fudge rounds even though Anthony possess an Absolute Advantage in the production of fudge rounds for ________. A. 𝐹𝐹 < 100 B. 100 < 𝐹𝐹 < 300 C. 𝐹𝐹 > 300 D. None of the above answers are correct, since there are no values of 𝐹𝐹 for which for which Oliver possess a Comparative A

C. 4 = 300 ÷ 75 Explanation: Let's break down the concepts of opportunity cost, absolute advantage, and comparative advantage in this scenario. Opportunity Cost: Opportunity cost is the value of the next best alternative given up when a choice is made. In this case, Anthony's opportunity cost for producing a unit of pork is the number of units of fudge rounds he could have produced instead Absolute Advantage: An individual or entity has an absolute advantage in the production of a good if they can produce more of that good with the same amount of resources or produce the same amount of the good with fewer resources. Comparative Advantage: Comparative advantage is based on the concept of lower opportunity cost. An individual or entity has a comparative advantage in the production of a good if the opportunity cost of producing that good is lower than that of another individual or entity. Anthony's Opportunity Cost = Units of Fudge Rounds / Units of Pork B. 100 < 𝐹 < 300 Explanation?

For Question 10, consider the market for Good X. The table below summaries the current market shares of the 8 largest producers of this good. (2ND HALF HW) Based upon only the values reported in the table above, A. C4 is equal to 45 and HHI is equal to 533. B. C4 is equal to 63 and HHI is equal to 619. C. C4 is equal to 45 but HHI cannot be calculated. D. neither C4 nor HHI can be calculated.

C. C4 is equal to 45 but HHI cannot be calculated. Explanation: The four-firm concentration ratio (C4) is calculated as follows. The four-firm concentration ratio (C4)=Sum of the market shares of the top 4 firms=15+12+10+8=45 So, The four-firm concentration ratio (C4) is 45. The HHI calculation needs data on market shares for all the firms in the market. This data is not given. The market shares of only 8 firms are given. It does not comprise 100% of the market. So, the HHI value cannot be calculated. Hence, The correct answer is alternative C where the C4 is 45, but the HHI value for the market cannot be calculated.

When discussing the "Three Fundamental Economic Questions," the ______________ asked, "Who gets to consume which goods and services?" A. Production Decision B. Resource Use Decision C. Distributional Decision D. Revenue Maximization Decision

C. Distributional Decision explanation: 1. What to produce? (production decision) 2. How to produce? (resource use decision) 3. For whom to produce? (distribution decision)

Which of the following was discussed in lecture to illustrate how something akin to the "Coasian Solution to Externalities" has been implemented in practice? A. How people with access to a common resource/good will tend to "overuse" it (relative to the efficient level of use), because benefits are concentrated to the user while costs are spread over everyone. B. How kids at "Chuck E. Cheese's" will often try to ride the carousel for free after another child pays a token to start the ride. C. How, in 2016, condo owners in a New York City apartment building voluntarily paid real estate developer Gary Barnett $11 million to NOT build a new high rise which would have blocked their views of the Empire State Building. D. How deliberately creating different version of a base product can increase profit for a seller, even if the different versions are costly to produce or are viewed by all consumers as b

C. How, in 2016, condo owners in a New York City apartment building voluntarily paid real estate developer Gary Barnett $11 million to NOT build a new high rise which would have blocked their views of the Empire State Building. Explanation: Coasian Solution to Externalities: This solution states that externalities can be efficiently managed through private negotiations between affected parties, without the need for government intervention. Akin to the Coasian Solution: The scenario in option C involves a negotiated agreement between parties affected by an externality (blocked view of the Empire State Building). The condo owners paid the developer to avoid the negative externality, similar to how parties might negotiate compensation to address other negative externalities.

In a perfectly competitive market A. there are many sellers but only one single buyer. B. firms have significant control over the price of their output (i.e., they are "price makers," not "price takers"). C. there is free entry/exit, so that in the Long Run new firms can enter and existing firms can exit the market with relative ease and without incurring substantial costs of doing so. D. More than one of the above answers is correct (which covers, but is not limited to, the case of "all of the above answers are correct").

C. there is free entry/exit, so that in the Long Run new firms can enter and existing firms can exit the market with relative ease and without incurring substantial costs of doing so. Explanation: A perfectly competitive market is recognized where neither seller or buyer can influence the price, there are no barriers to entry / exit, prices are same at all levels of output and Average revenue and Marginal revenue are equal and there are large no of buyers and sellers.

For Questions 7 through 9, consider a society with three workers (Harry, Mitch, and Tyler) who spend their workday producing two goods (watermelon and sugar). Their daily "piecewise linear" Production Possibilities Frontier (i.e., three "straight-line" segments) is illustrated below. Points on the frontier between "B" and "C" are obtained as Harry switches his time away from production of sugar and toward production of watermelons. -. (1ST HALF HW) Which of the following combinations of output is unattainable? A. Output combination "A" (160 units of watermelons and 260 units of sugar). B. Output combination "F" (230 units of watermelons and 260 units of sugar). C. Output combination "G" (230 units of watermelons and 400 units of sugar). D. None of the above answers are correct.

C. Output combination "G" (230 units of watermelons and 400 units of sugar). Explanation: Output combination "A" (160 units of watermelons and 260 units of sugar) is attainable, as it is on the PPF. Output combination "F" (230 units of watermelons and 260 units of sugar) is also attainable, as it is on the PPF. However, output combination "G" (230 units of watermelons and 400 units of sugar) is unattainable, as it is outside the PPF. Therefore, the answer to the question is C. Output combination "G" (230 units of watermelons and 400 units of sugar).

For Question 6, consider a monopolist facing demand as illustrated below and with marginal revenue and costs as illustrated below. (HW7) When this monopolist chooses price and quantity to maximize profit, __________________________. A. Consumers' Surplus is equal to areas a+b+c+d+e+f B. Deadweight Loss is equal to areas f+j C. Producer's Surplus is equal to areas c+d+g+h D. None of the above answers are correct.

C. Producer's Surplus is equal to areas c+d+g+h Explanation: As per given diagram: i) Given, Monopolist firms, Revenue, Cost and Demand Curves. ii) profit maximizing output is determined with the help of point of intersection of marginal revenue and marginal cost curves. Therefore, the Optimal Output is 460 units and Explanation: ii)The vertical line from Q is "460" to intercept the Demand curve is the point to determine the Optimal Price. Therefore, Optimal level of Price is. " 16.25" iii) In case of Monopoly, Consumer Surplus is measured as the Area of ,(a+b+c+d) iv) Producer Surplus is Measured as the Area of ( c+ d+ g+ h)

WBC Heavyweight Champion Tyson Fury (aka, The Gypsy King) last defended his title on December 3, 2022 when he came out of retirement to fight Derek Chisora (who does not have a cool nickname) at Tottenham Hotspur Stadium in London, England. Fury won this fight by technical knockout in the tenth round. The broadcast of this fight in the U.S. on ESPN+ was excludable and non-rival in consumption, and was therefore A. an inferior good. B. a public good. C. a club good. D. a common good.

C. a club good. Explanation?

Economics can be defined as the social science that studies how people make decisions in the face of scarcity. "Scarcity" refers to A. the positive difference between quantity supplied and quantity demanded at prices above the market equilibrium price. B. the observation that demand will decrease if consumer income decreases. C. a universal phenomenon that arises because resources are finite and limited. D. the recognition that society can increase its output of all goods if resources are used to produce the outputs for which they possess an Absolute Advantage.

C. a universal phenomenon that arises because resources are finite and limited. Explanation: Scarcity is the foundational concept of economics because it shapes how individuals, businesses, and governments make choices in a world of limited resources and unlimited wants. It is the driving force behind economic decision-making, resource allocation, and the study of economics as a social science. A. Scarcity is not about the difference between quantity supplied and quantity demanded at prices above the market equilibrium price. This statement describes a situation related to market dynamics and not the fundamental concept of scarcity. B. Scarcity is not solely about the observation that demand will decrease if consumer income decreases. This statement is related to how changes in consumer income affect demand, but it does not capture the broader concept of scarcity, which is rooted in the limitation of resources. D. Scarcity is not about the recognition that society can increase its output of all goods if resources are used to produce the outputs for which they possess an absolute advantage. This statement discusses the concept of absolute advantage, which is related to international trade theory, but it does not directly address the fundamental concept of scarcity.

Consider the market for wheat. Suppose that between April 2023 and September 2023 there was a decrease in equilibrium price but increase in equilibrium quantity. This change would have resulted from: A. an increase in demand (with no change in supply). B. a decrease in demand (with no change in supply). C. an increase in supply (with no change in demand). D. a decrease in supply (with no change in demand).

C. an increase in supply (with no change in demand). Explanation: If there was a decrease in equilibrium price but an increase in equilibrium quantity in the market for wheat between April 2023 and September 2023, this change would have resulted from an increase in supply (with no change in demand). A decrease in equilibrium price suggests that the price at which wheat is being sold in the market has gone down during this period. An increase in equilibrium quantity means that the quantity of wheat being bought and sold in the market has gone up during this period. When there's an increase in supply (with no change in demand), it means that more wheat is being produced and offered for sale in the market, which can lead to a decrease in price due to increased availability. At the same time, since the demand hasn't changed, people are buying more wheat because it's more affordable, leading to an increase in quantity. I hope this helps clarify the answers! If you have any more questions or need further clarification, feel free to ask.

The "Tragedy of the Commons" illustrates A. how people will attempt to enjoy the benefits of units of a common good purchased by others, while not purchasing any units themselves. B. how First Degree Price Discrimination generally leads to an increase in Deadweight Loss. C. why a common good will be used/consumed by members of society at a level above the socially best level. D. why a common good will be used/consumed by members of society at a level below the socially best level.

C. why a common good will be used/consumed by members of society at a level above the socially best level. Explanation?

The Efficient Scale of Production refers to the quantity of output at which A. the profit of the firm is maximized. B. Fixed Costs are minimized. C. Total Costs are minimized. D. Average Total Costs are minimized.

D. Average Total Costs are minimized. Explanation: Efficient Scale of Production is the level of production where the average total costs are at their lowest point. At this point, the firm maximizes its efficiency for producing goods.

For Questions 3 and 4, consider the market illustrated below. Demand for this good is linear. Recognize that the slope of this demand curve is equal to -1/3,500 . (refer to hw3 no 3) At a price of $11.30, quantity demanded is 16,450 units. Consequently, the value of price elasticity of demand is A. approximately - 0.416. B. approximately - 1.988. C. approximately - 2.404. D. - 4.7. If an increase in supply led to a decrease in price from $5.36 to $4.87, total consumer expenditures on this good would A. increase. B. decrease. C. remain constant (i.e., not change). D. None of the above answers are necessarily correct, since the graph does not convey enough information to determine how total consumer expenditures would change for this decrease in price.

C. approximately - 2.404. Explanation: Elasticity of demand is given by: (1/slope) x price/quantity Using above formula and putting the values, we have Given slope= -1/3500 Now; Ed= -3,500 x 16,450/11.30 =-2.404 B. decrease. Explanation: The demand curve is expressed as Q=a+bP. Here b is the slope which is -3500. When Q is 56000, P is 0. This implies that 56,000=a+0×−3,500 which suggests that a is 56000. Now the demand equation is Q=56,000−3,500P Explanation: When price changes from 5.36 to 4.87, the quantity changes from 37240 units to 38955 units. The expenditure changes from 5.36×37,240=199,606.40 to 38,955×4.87=189,710.9. Hence, the expenditure has reduced.

For questions 19 and 20 consider the market illustrated below. The efficient quantity of trade is 2,520 units. (1st half hw) If a Price Floor of $16.25 is imposed in this market there would be a Deadweight Loss equal to A. areas a+b. B. areas a+b+c+d. C. areas c+d. D. area e. Suppose a per unit subsidy of $6.00 is paid to buyers. The quantity traded with this subsidy in place would be A. 1,595 units. B. more than 1,595 but less than 2,520 units C. more than 2,520 but less than 3,950 units. D. 3,950 units.

C. areas c+d. Explanation? C. more than 2,520 but less than 3,950 units. Explanation?

Normative Statements A. are fact based statements. B. can potentially be confirmed or refuted by gathering sufficient evidence and data. C. attempt to assess the desirability of how the world is or functions, often with suggestions of things that could be done to improve outcomes. D. attempt to describe how the world actually is or actually functions.

C. attempt to assess the desirability of how the world is or functions, often with suggestions of things that could be done to improve outcomes. Explanation: Normative Statements are opinion based or "valued base" statements. A normative statement is an expression that something is right or wrong so often includes the words ought, should or better.

Answer Questions 5 and 6 based upon the information conveyed in the following table, which states Erin's Total Benefits from consumption of "Good Y" each month (measured in dollars). (refer to HW1 NO.5 FOR PIC) Suppose that Erin has the option to purchase Good Y be either: (i) paying $15 for each unit consumed or (ii) paying a flat fee $150 to be able to consume as much as she wants (for example, maybe this good is access to a fitness center and she can pay $15 for each visit or pay a flat fee of $150 per month which allows her to visit as many times as she wants for no additional charge). Given these pricing options, Erin should A. choose to pay $15 for each unit consumed and get 9 units of Good Y. B. choose to pay $15 for each unit consumed and get 10 units of Good Y. C. choose to pay a flat fee of $150 and get 12 units of Good Y. D. choose to pay a flat fee of $150 and get 13 units of Good Y.

C. choose to pay a flat fee of $150 and get 12 units of Good Y. Explanation: Case 1: If Erin chooses to pay $15 ---------------------------------------- The Marginal benefit of the 9th unit is $16 The Marginal benefit of the 10th unit is $10 While Marginal cost is $15 As seen above, Erin would consume only 9 units because MB is higher than the MC Erins Net Benefit = Total Benefit - Total Cost =346 - 9 x 15 =346 - 135 =211 Hence Erin's Net Benefit would be 211$ in this option Case 2: If Erin chooses to pay a flat fee of $150 ---------------------------------------------------- The Total cost of Erin is fixed at $150 So, Net benefits are maximized when Total benefits are maximum So, Erin would consume 12 units where the Total Benefit is the maximum Erin's net benefit= Total Benefit - Total Cost 364 -150 =214 Hence Erin's Net Benefit would be $214 in this option As seen from the above calculations, Erin's Net benefits are Higher when she pays a flat fee of $150 and consumer 12 units of Y

A "rival" good is a good for which A. intense competition between competing producers drives price down below marginal costs of production. B. a seller is able to charge every consumer a price exactly equal to the their reservation price for the good. C. consumption by one person diminishes the quantity/quality of consumption by others. D. consumption by one person does not diminish the quantity/quality of consumption by others.

C. consumption by one person diminishes the quantity/quality of consumption by others. Explanation?

For Question 8, consider the following scenario. Johnny is the only seller of "Good X" in Kennesaw, GA. Demand and Marginal Costs for his output are illustrated in the graph below. Marginal Costs of production are minimized if he produces 6,000 units. Suppose Johnny is able to engage in First Degree (i.e., Perfect) Price Discrimination. (hw10) When Johnny maximizes profit by way of engaging in Perfect Price Discrimination A. his Profit is $44,000, Total Consumers' Surplus is $27,000, and Deadweight Loss is $54,000. B. his Profit is $77,000, Total Consumers' Surplus is $108,000, and Deadweight Loss is $0. C. his Profit is $125,000, Total Consumers' Surplus is $0, and Deadweight Loss is $0. D. his Profit is $185,000, Total Consumers' Surplus is $0, and Deadweight Loss is $0.

C. his Profit is $125,000, Total Consumers' Surplus is $0, and Deadweight Loss is $0. Explanation?

For Questions 7 and 8, consider the market for Good X. The table below summaries the current market shares of different producers of this good, along with projected market shares two years from now if a proposed merger between two existing firms takes place. (HW7) 7. If this merger takes place, then (according to the projected market shares in 2025), this market will become A. less competitive (i.e., closer to monopoly) according to C4, but more competitive (i.e., further from monopoly) according to HHI. B. less competitive (i.e., closer to monopoly) according to HHI, but more competitive (i.e., further from monopoly) according to C4. C. less competitive (i.e., closer to monopoly) according to both C4 and HHI. D. more competitive (i.e., further from monopoly) according to both C4 and HHI.

C. less competitive (i.e., closer to monopoly) according to both C4 and HHI. Explanation: For 2023: C4 is the combined market share of the four largest firms. C4=20+16+14+12=62% HHI is the sum of the squares of the market shares of all firms. HHI=202+162+142+122+112+102+82+72+22=400+256+196+144+121+100+64+49+4=1,334 For 2025 (projected): C4 for 2025 =30+15+13+12=70% HHI for 2025 =302+152+132+122+112+102+82+52+22=900+225+169+144+121+100+64+25+4=1,752 Answer to Question 7: The C4 value increases from 62% to 70%, indicating the market is becoming less competitive (closer to monopoly). The HHI value increases from 1334 to 1752, which also indicates the market is becoming less competitive. Thus, the answer is: C. less competitive (i.e., closer to monopoly) according to both C4 and HHI.

Consider a firm operating in a perfectly competitive market in the short run with costs such that the minimum value of Average Variable Costs is $12.97 and the minimum value of Average Total Costs is $14.23. If the price of the output of the firm is $13.56 per unit, then the firm would choose to A. shut down in the short run and earn zero profit. B. shut down in the short run and earn a negative profit. C. produce a positive quantity of output in the short run but earn a negative profit. D. produce a positive quantity of output in the short run and earn a positive profit.

C. produce a positive quantity of output in the short run but earn a negative profit. Explanation: The firm's minimum average variable cost (AVC) is $12.97, and its minimum average total cost (ATC) is $14.23. The price of the firm's output is $13.56 per unit. In a perfectly competitive market, a firm should continue to produce in the short run if the price it receives for its product (P) is greater than its average variable cost (AVC), because it will cover all its variable costs and there will be some contribution to fixed costs. Here, the price of $13.56 is above the minimum AVC of $12.97, which means the firm can cover all variable costs and have a contribution of $13.56 - $12.97 = $0.59 per unit towards its fixed costs. However, since the price is below the ATC, the firm will not make a profit. Thus, the firm will: Produce a positive quantity of output in the short run (because P > AVC). Incur a loss in the short run (because P < ATC).

For Question 3 and 4, consider a firm operating in a Monopolistically Competitive market in the Short Run facing demand as illustrated below and with marginal revenue and costs as illustrated below. (HW8) 3. To maximize Short Run Profit, this firm should A. sell 1,750 units and charge a price of $5.40. B. sell 1,750 units and charge a price of $13.10. C. sell 1,750 units and charge a price of $17.05. D. sell 2,780 units and charge a price of $10.95.

C. sell 1,750 units and charge a price of $17.05. Explanation: In the short run a firm in a monopolistically competitive market would maximize its profit by operating at a point where marginal revenue equals marginal cost. Here marginal revenue equals marginal cost at Q=1750 units, the price charged by the firm is the price corresponding to Q=1750 on the demand curve. Therefore the price is 17.05. Hence the firm will sell 1750 units at a price of 17.05 per unit. Q*=1750 P*=17.05

A Price Ceiling will typically make A. all buyers and all sellers worse off. B. all buyers and all sellers better off. C. some buyers better off and some buyers worse off, but all sellers worse off. D. some sellers better off and some sellers worse off, but all buyers better off

C. some buyers better off and some buyers worse off, but all sellers worse off Explanation: Price ceilings are upper bounds on the cost of specific commodities or services set by the government. By keeping costs lower than they would be in a free market, they are made to benefit consumers. However, they frequently result in shortages, which can either benefit some customers (who can buy the product in short supply at a lower price) or hurt certain customers (who can't find it because of shortages). Because they are unable to sell at higher market prices, all sellers are suffering in the meanwhile. Price ceilings are upper bounds on the cost of specific commodities or services set by the government. By keeping costs lower than they would be in a free market, they are made to benefit consumers. Nevertheless, they frequently cause shortages and can make some customers worse off (those who cannot find the goods owing to shortages) and some customers better off (those who can acquire the limited supply at the lower price). Because they are unable to sell at higher market prices, all sellers are suffering in the meanwhile.

For question 6 consider a situation in which a seller is considering offering multiple versions of a product to two different types of consumers (High Income and Middle Income). The table below states reservation prices of each consumer type for each version. Each consumer will either buy one item (the version which gives her the largest, non-negative consumer's surplus) or not buy at all (if all purchase options would give negative consumer's surplus). The values in the bottom row state the number of each consumer type present in the market (i.e., there are 40,000 High Income and 60,000 Middle Income consumers). (HW11) Suppose the seller has marginal costs of $1 for each customer served, regardless of which version the customer purchases. The seller has zero Fixed Costs, other than (potentially) a Quasi-Fixed Cost of creating value subtracted versions as follows. If the seller wants to offer the "Low Quality

C. the Standard Version and Very Low Quality Version. Explanation?

In a perfectly competitive market, A. there is only one single seller. B. there are many different sellers, each selling products that are differentiated (i.e., different) from those being sold by other sellers. C. there are many different sellers, each selling products that are identical to those being sold by others sellers. D. there are not any sellers (only buyers).

C. there are many different sellers, each selling products that are identical to those being sold by others sellers Explanation: All the other options are Incorrect. A perfectly competetive market is recognized where neither seller or buyer can influence the price, there are no barriers to entry / exit, prices are same at all levels of output and Average revenue and Marginal revenue are equal and there are large no of buyers and sellers.

For Questions 6 and 7, consider the market with Demand given by the inverse demand function 𝑃𝐷(𝑞𝑞) = 45,000/𝑞 and Supply given by the inverse demand function 𝑃𝑆(𝑞) = 8 + 1/600 𝑞 . The market equilibrium outcome (which is efficient) is approximately 𝑝∗ ≈ 13.54 and 𝑞∗ ≈ 3,324. Consider the following two per unit taxes. Tax I: a $3 tax imposed on buyers. Tax II: a $4 tax imposed on sellers. Which of the following statements is correct? A. "The quantity traded of the good would be smaller under Tax I than under Tax II." B. "Of these two options, buyers would prefer for the government to impose Tax II." C. "Of these two options, sellers are indifferent over which one the government imposes." D. "Deadweight Loss would be larger under Tax II than under Tax I."

D. "Deadweight Loss would be larger under Tax II than under Tax I." Explanation: Option: (D) It does not matter whom the tax is imposed on. The burden of tax is shared between buyer and seller based on the prevailing elasticities of demand and supply. Incorrect Option Option (A) is incorrect because tax I is larger than Tax II, thus quantity traded would be larger in case of Tax I. Option (B) is incorrect because it does not make difference whom the tax is imposed on. Option (C) is incorrect because deadweight loss is larger in case of Tax II. The seller will prefer the Tax I.

According to the Inverse Elasticity Pricing Rule, to maximize profit a firm must charge a price and sell a quantity of output for which −1/𝜀𝑝 is equal to ____________. A. price (i.e., 𝑝) B. "price/marginal cost margin" or the difference between price and marginal cost (i.e., 𝑝 − MC) C. the ratio of marginal cost to price (i.e., MC/𝑝 ) D. "price/marginal cost margin as a percentage of price" or the difference between price and marginal cost divided by price (i.e., 𝑝−MC/𝑝 )

D. "price/marginal cost margin as a percentage of price" or the difference between price and marginal cost divided by price (i.e., 𝑝−MC/𝑝 ) Explanation?

For Questions 7 through 9, consider a society with three workers (Harry, Mitch, and Tyler) who spend their workday producing two goods (watermelon and sugar). Their daily "piecewise linear" Production Possibilities Frontier (i.e., three "straight-line" segments) is illustrated below. Points on the frontier between "B" and "C" are obtained as Harry switches his time away from production of sugar and toward production of watermelons. -. (1ST HALF HW) Mitch's Opportunity Cost for producing a unit of watermelons is ________ units of sugar. (As noted above, Mitch is the "marginal worker" between point "C" and point "D" on the Production Possibilities Frontier.) A. 0.4 B. 0.5 C. 0.75 D. 2

D. 2 Explanation: To find the opportunity cost of any good X in terms of the units of Y given up, we use the following formula: Opportunity cost of each unit of good X=Y1−Y2/X1−X2 Mitch's opportunity cost for producing a unit of watermelons is 400−260/160−230=140/−70=−2

Consider a perfectly competitive market in the Short Run in which there are • 30 firms with MC(𝑞) = 1/500𝑞 + 3 and AVCmin = 3 • 10 firms with MC(𝑞) = 1/2,000𝑞 + 4 and AVCmin = 4 • 20 firms with MC(𝑞) = 1/1,750𝑞 + 6 and AVCmin = 6 At a price of $5, market quantity supplied would be equal to ________ units. A. 130.325 B. 1,250 C. 3,000 D. 50,000

D. 50,000 Explanation: For each firm, we can set the MC equal to the market price ($5) to find the quantity supplied by each firm: Explanation: The quantity supplied in a perfectly competitive market is determined by the firms' marginal cost (MC) and the market price. The market quantity supplied is the sum of individual quantities supplied by each firm at the given price. 1. For the first 30 firms: MC1​(q)=1/500​q+3=5 Solve for q1​: 1/500​q+3=5 1/500q1​=2 q1​=1,000 Step 2 For the next 10 firms: MC2​(q)=1/2,000​q+4=5 Solve for q2​:1/2,000​q2​+4=5 1/2,000q2​=1 q2​=2,000 3. For the last x20 firms: MC3​(q)=1/1,750q+6=5 Solve for q3​:1/1,750q3​+6=5 1/1,750​q^3​=−1(This implies that q3​ is negative, which doesn't make sense in this context.) Now, let's calculate the total quantity supplied: Q=30×q1​+10×q2​ Q=30×1,000+10×2,000=30,000+20,000=50,000 Explanation: So, the correct answer is D.50,000 units.

Which of the following could never cause a change in demand for Good X? A. A change in the income of consumers of Good X. B. A change in the weather. C. A change in the price of Good Y. D. A change in the price of Good X.

D. A change in the price of Good X. Explanation: The demand curve for good X represents the relationship between the price of good X and the quantity demanded of good X, holding all other factors constant. Therefore, a change in the price of good X itself will cause a movement along the demand curve, not a shift of the demand curve. On the other hand, changes in the price of other goods (complementary or substitute goods) or changes in average disposable real income can cause the demand curve for good X to shift. To summarize: A change in the price of x (good X itself) will cause a movement along the demand curve, not a shift. A change in the price of y (a complementary good) can cause the demand curve for good X to shift. A change in the price of z (a substitute good) can cause the demand curve for good X to shift. An increase in average disposable real income can cause the demand curve for good X to shift.

For Question 8, consider a situation in which two duopolists compete by simultaneously choosing levels of output, 𝒒𝑨 and 𝒒𝑩. Firm A has constant marginal costs of 𝒄𝑨 = 39, and Firm B has constant marginal costs of 𝒄𝑩 = 𝟑0. Market demand is given by the inverse function 𝑷𝑫(𝑸) = 180 − 𝑸. Which of the following statements about the equilibrium of this game is true? "In equilibrium,... A. both firms produce the same amount of output and earn the same amount of profit." B. both firms produce the same amount of output but Firm B earns a larger profit than Firm A." C. Firm A produces more output than Firm B but Firm B earns a larger profit than Firm A." D. Firm B produces more output than Firm A and Firm B earns a larger profit than Firm A."

D. Firm B produces more output than Firm A and Firm B earns a larger profit than Firm A." Explanation?

For Question 8, consider a monopolist (who cannot engage in any sort of Price Discrimination) facing demand and with marginal revenue and costs as illustrated below. (2ND HALF HW) When this monopolist chooses price and quantity to maximize profit, __________________________. A. Consumers' Surplus is equal to areas a+b B. Deadweight Loss is equal to areas h+i+j+k C. Producer's Surplus is equal to areas c+d+e+f+g D. More than one of the above answers is correct (which covers, but is not limited to, the case of "all of the above answers are correct")

D. More than one of the above answers is correct (which covers, but is not limited to, the case of "all of the above answers are correct"). [A and B are both correct] Explanation: Profit maximisation condition is where the marginal cost is equal to marginal revenue. Marginal cost is equal to marginal revenue when the Quantity is equal to 230 and Price is equal to 48.75 Consume surplus is area below demand curve and above price. Consumer surplus is equal to area a + b. Producer surplus is area above supply curve (rising part of marginal cost) and below price. Producer surplus is equal to area c + d + e + f . Deadweight loss is equal to loss in total surplus. Deadweight loss is equal to h + i + j + k The profit maximisation condition is where the marginal cost is equal to the marginal revenue. Consume surplus is the area below the demand curve and above price. Producer surplus is the area above the supply curve (rising part of the marginal cost) and below price. Deadweight loss is equal to loss in total surplus.

Focusing on Price Elasticity of Demand, in general, Demand will be "more elastic" for goods A. that are "luxuries" (as opposed to "necessities"). B. on which consumers spend a "large share" (as opposed to a "small share") of their total budget. C. for which there are "many close substitutes" (as opposed to "very few close substitutes"). D. More than one of the above answers is correct (which covers, but is not limited to, the case of "all of the above answers are correct").

D. More than one of the above answers is correct (which covers, but is not limited to, the case of "all of the above answers are correct"). [A, B, & C are all correct] Explanation: A. Demand will be "more elastic" for goods that are "luxuries" (as opposed to "necessities"). This statement is generally correct. Goods that are considered luxuries tend to have more elastic demand because consumers are more responsive to price changes for non-essential items. When the price of a luxury item goes up, consumers are more likely to reduce their consumption or switch to alternatives B. Demand will be "more elastic" for goods on which consumers spend a "large share" (as opposed to a "small share") of their total budget. This statement is generally correct as well. When a good represents a large portion of a consumer's budget, they are more sensitive to price changes for that good. Therefore, the demand for goods that represent a significant portion of a consumer's budget tends to be more elastic. C. Demand will be "more elastic" for goods for which there are "many close substitutes" (as opposed to "very few close substitutes"). This statement is also generally correct. When there are many close substitutes available for a product, consumers have more options to switch to if the price of the product increases. This makes the demand for the product more elastic because consumers can easily switch to alternatives. D. More than one of the above answers is correct (which covers, but is not limited to, the case of "all of the above answers are correct"). All of the above answers (A, B, and C) are correct, and they all describe conditions that tend to make demand more elastic.

The Variable Costs of producing Q=5,000 units of output can be graphically illustrated by A. the entire area below the Marginal Cost Curve from Q=0 units of output up to Q=5,000 units of output. B. the area of the rectangle from the point on the Average Fixed Costs Curve at Q=5,000 back to the origin. C. the area of the rectangle from the point on the Average Variable Costs Curve at Q=5,000 back to the origin. D. More than one of the above answers is correct (which covers, but is not limited to, the case of "all of the above answers are correct").

D. More than one of the above answers is correct (which covers, but is not limited to, the case of "all of the above answers are correct"). [Both "A" and "C" are correct.] Explanation: Option D is correct because both the area below the marginal cost curve and the area of the rectangle from the point on the average variable costs curve at Q=5,000 back to the origin can be used to graphically illustrate the variable costs of producing Q=5,000 units of output. Option C: This option is correct because the area of the rectangle from the point on the average variable costs curve at Q=5,000 back to the origin represents the total variable cost of producing 5,000 units of output. Option A: This option is correct because the area below the marginal cost curve, up to a given quantity of output, represents the total variable cost of producing that quantity of output. Therefore, the entire area below the marginal cost curve from Q=0 units of output up to Q=5,000 units of output represents the total variable cost of producing Q=5,000 units of output. Option B: This option is not correct because the average fixed costs curve shows the total fixed cost per unit of output. The total variable cost of producing a given quantity of output is not related to the average fixed cost per unit of output.

For Question 5, consider a game in which the "Best Response Arrows" are as illustrated below: (HW8) Based upon the Best Response Arrows above A. this game fits the definition of a Prisoner's Dilemma. B. this game has multiple Nash Equilibria. C. Player 1 has a dominant strategy, but Player 2 does not. D. Player 2 has a dominant strategy, but Player 1 does not.

D. Player 2 has a dominant strategy, but Player 1 does not. Explanation: When Player 2 chooses strategy Y: - Player 1's best response to Y is W (indicated by the left arrow). - When Player 2 chooses strategy Z: - Player 1's best response to Z is X (indicated by the right arrow). From the above observations: - Player 1 doesn't have a strategy that is a best response irrespective of what Player 2 chooses, so Player 1 doesn't have a dominant strategy. Player 2's best response to both of Player 1's strategies (W and X) is Y, so Y is a dominant strategy for Player 2. Given these findings, the correct answer is: D. Player 2 has a dominant strategy, but Player 1 does not.

________________ refers to the practice of selling multiple distinct products together as a package, while NOT allowing customers the option to purchase the items outside of the package. A. 3rd Degree Price Discrimination (or "Segmented Pricing") B. Versioning C. Mixed Bundling D. Pure Bundling

D. Pure Bundling Explanation?

Answer Questions 3 and 4 based upon the information conveyed in the following graph (which illustrates two different levels of Marginal Benefits of an activity for a decision maker). Suppose throughout that the Marginal Costs of the activity are equal to a constant $6.95. (1ST HALF HW) Suppose Marginal Benefits are given by MB1 (and Marginal Costs are MC=$6.95). If the decision maker increased Q from Q=1,320 to Q=1,540, then A. Total Costs would increase, but Total Benefits and Total Economic Surplus would both decrease. B. Total Costs would remain constant, Total Benefits would decrease, and Total Economic Surplus would decrease. C. Total Costs and Total Benefits would both increase, but Total Economic Surplus would decrease. D. Total Costs, Total Benefits, and Total Economic Surplus would all increase.

D. Total Costs, Total Benefits, and Total Economic Surplus would all increase. Explanation: If decision maker increased Q from 11 = 1320 to Q2= 1540. The decision maker is increasing the Q in the direction of profit maximizing quantity i.e = 2150 ( Quantity where marginal benefit is equal to marginal cost) Then, 1) the total cost increases because quantity is increasing . 2) Total benefit also increases because Marginal benefit is falling , but its positive. 3)Total economic surplus will also increase because it the quantity is moving towards profit maximizing quantity which is an efficient point. Hence answer D.

For Questions 4 and 5, consider a situation in which a seller is offering three different versions of a product (X, Y, and Z) to three different consumer types (A, B, and C). The table below states reservation prices of each consumer type for each version, along with marginal costs of production for each version. Each consumer will either buy one item (the version which gives her the largest, non-negative consumer's surplus) or not buy at all (if all purchase options would give negative consumer's surplus). The values in the bottom row state the number of each consumer type present in the market (e.g., there are 1,000 consumers of Type B). (HW11) For these different version, there appears to be ____________ Product Differentiation. A. Diagonal B. Horizontal C. Marginal D. Vertical Of the four sets of prices below, which would give the seller the most profit? A. Charge $24 for Version X, $25 for Version Y,

D. Vertical Explanation? B. Charge $31 for Version X, $17 for Version Y, and $8 for Version Z Explanation?

Market Failure is defined as A. a complete ban on an activity that generates a negative externality B. a subsidy payment made to a seller who produces a good that generates a positive externality. C. a situation in which a seller has control over the price of its output (in that it can generally increase price a little without quantity demanded going to zero). D. a situation in which the free market outcome is inefficient, in that Deadweight Loss is positive at the resulting free market level of trade

D. a situation in which the free market outcome is inefficient, in that Deadweight Loss is positive at the resulting free market level of trade Explanation?

Consider the market for corn. Suppose that between May 2023 and August 2023 there was an increase in demand and decrease in supply. It follows that between these two dates: A. equilibrium quantity must have decreased, but equilibrium price could have increased, decreased, or remained unchanged. B. equilibrium quantity must have increased, but equilibrium price could have increased, decreased, or remained unchanged. C. equilibrium price must have decreased, but equilibrium quantity could have increased, decreased, or remained unchanged. D. equilibrium price must have increased, but equilibrium quantity could have increased, decreased, or remained unchanged.

D. equilibrium price must have increased, but equilibrium quantity could have increased, decreased, or remained unchanged. Explanation?

"Nash's Existence Theorem" states that A. a unique per unit tax which leads to the efficient level of production in the face of a negative externality (known as a Nash Tax) always exists. B. when selling a good by way of Segmented Pricing, a unique pair of prices (known as Nash Prices) always exists which makes the seller better off and makes all consumers better off than they would be if the firm instead had to engage in simple monopoly pricing. C. in every two-player game in which each player has two available strategies, a strategy (known as a Nash Strategy) always exists for each player which gives her a higher payoff than her other strategy, regardless of the choice of her rival. D. for every game with any finite number of players, each with a finite number of available pure strategies, there exists at least one Nash Equilibrium (potentially in "mixed strategies").

D. for every game with any finite number of players, each with a finite number of available pure strategies, there exists at least one Nash Equilibrium (potentially in "mixed strategies"). Explanation: Consequently, option D appropriately captures the importance and universality of Nash's Existence Theorem. Regardless of the particular game structure or player objectives, it ensures that every finite game with finite pure strategies has at least one Nash equilibrium. As a result, option D provides the most accurate and thorough explanation of the theorem, emphasizing its critical importance in comprehending strategic behavior and forecasting results in a range of real-world situations.

For Question 8, consider the market illustrated below. The efficient quantity of trade is 1,550 units. (HW4) Suppose that the government pays producers of this good a per unit subsidy of $3.35. The quantity traded with this subsidy in place would be __________. A. greater than 610 but less than 1,550 B. greater than 1,550 but less than 2,090 C. exactly 2,090 D. greater than 2,090 but less than 2,750

D. greater than 2,090 but less than 2,750 Explanation: The demand curve is the downward sloping curve. Supply curve is the upward sloping curve. Equilibrium is where the demand curve intersects the supply curve. The imposition of tax on seller shifts the supply curve to the left. The tax burden is calculated using the new price paid by the consumer, new price received by the producer and the equilibrium price before tax. The subsidy to producer shifts the supply curve to the right. As the government provides subsidy of $3.35 to producers, The supply curve will shift to right by $3.35 at each quantity. So, The new equilibrium quantity will be greater than the previous equilibrium quantity (1550) as there is a right shift of the supply curve. So, The quantity traded will be greater than 1550. At a quantity of 2090, The willingness to pay of consumers = $1.70 And the willingness to accept of producers = $4 - $3.35 = $0.65 As the WTP is greater than the WTA, the more quantity will be traded upto when the WTP is equal to WTA. As the increase in quantity will decrease the WTP and increase the WTA. So, The quantity traded will be greater than 2090 At a quantity of 2750, The willingness to pay of consumers is less than $1.70 And the willingness to accept of producers = $6.30 - $3.35 = $2.95 As the WTA is greater than the WTP, the less quantity will be traded upto when the WTP is equal to WTA. As the decrease in quantity will decrease the WTA and increase the WTP. So, The quantity traded will be less than 2750 Therefore, The correct option is (d). Greater than 2090 but less than 2750. All the remaining options are incorrect as per the above calculations.

Consider a firm operating in a perfectly competitive market in the Short Run. For this firm, the minimum value of Average Variable Costs is $38.27 and the minimum value of Average Total Costs is $42.19. Fixed Costs of Production are $7,800. If the per unit price of the output of this firm is $44.37, then the firm will A. shut down and incur a loss of exactly $7,800. B. produce a positive quantity of output and incur a loss of exactly $7,800. C. produce a positive quantity of output and incur a loss of less than $7,800. D. produce a positive quantity of output and earn a positive profit.

D. produce a positive quantity of output and earn a positive profit. Explanation?

The value of Price Elasticity of Demand for Good X is -1.48. If price were to increase by 2.6%, it follows that A. quantity demanded would decrease by approximately 0.569% B. quantity demanded would decrease by approximately 1.120% C. quantity demanded would decrease by approximately 1.757% D. quantity demanded would decrease by approximately 3.848%

D. quantity demanded would decrease by approximately 3.848% Explanation: To calculate the percentage change in quantity demanded given the price elasticity of demand (-1.48) and a price increase of 2.6%, you can use the following formula: Percentage Change in Quantity Demanded = Price Elasticity of Demand * Percentage Change in Price Percentage Change in Quantity Demanded = (-1.48) * (2.6%) Percentage Change in Quantity Demanded ≈ -3.848% So, the correct answer is D. Quantity demanded would decrease by approximately 3.848%.

For Question 3 and 4, consider a firm operating in a Monopolistically Competitive market in the Short Run facing demand as illustrated below and with marginal revenue and costs as illustrated below. (HW8) 4. In the Long Run this firm would want to A. completely exit the industry. B. remain in the industry and should expect some of its current rivals to exit, causing a decrease in costs of production and an increase in profit. C. remain in the industry and should expect some new rivals to enter, causing an increase in costs of production and a decrease in profit. D. remain in the industry and should expect some new rivals to enter, causing a decrease in demand a

D. remain in the industry and should expect some new rivals to enter, causing a decrease in demand and a decrease in profit. Explanation: In the short run Q*=1750 and P*=17.05. Total revenue =P×Q Total revenue = 17.05×1,750 Total revenue = 29,837.5 At Q=1750, ATC or average total cost is equal to 13.10 Total cost = ATC×Q Total cost = 13.10×1,750 Total cost = 22,925 Profit = Total revenue - Total cost Profit = 6,912.5>0 Since the firm is earning profit earning in the short run it will remain in the industry. This profit will attract new rivals to enter the market as a result the residual demand faced by each firm will decrease leading to a decrease in profit. Hence the firm will remain in the industry and expect some new rivals to enter, causing a decrease in demand and a decrease in profit.

For Question 5, consider a firm operating in a perfectly competitive market with costs as partially summarized in the table below: (HW6) If this firm faces a per unit price of $25 for its output, then A. profit is maximized by shutting down in the Short Run. B. the firm should hire 1 worker but will earn a negative profit in the Short Run. C. the firm should hire 3 workers and can earn a positive profit in the Short Run. D. the firm should hire 5 workers and can earn a positive profit in the Short Run

D. the firm should hire 5 workers and can earn a positive profit in the Short Run. Explanation: The profit-maximizing level of output and the number of workers the firm should hire in the short run, we need to calculate the profit at each level of output and the corresponding number of workers. The profit is calculated as follows: Profit = Total Revenue - Total Cost Explanation: Total Revenue = Price per unit × Quantity of output Total Cost = Fixed Costs + Variable Costs Let's calculate the profit for each level of output: For 1 worker: Quantity of output = 50 units Total Revenue = $25 × 50 = $1250 Total Cost = $240 (Fixed Costs) + $30 (Variable Costs) = $270 Profit = $1250 - $270 = $980 For 2 workers: Quantity of output = 90 units Total Revenue = $25 × 90 = $2250 Total Cost = $480 (Fixed Costs) + $60 (Variable Costs) = $540 Profit = $2250 - $540 = $1710 For 3 workers: Quantity of output = 120 units Total Revenue = $25 × 120 = $3000 Total Cost = $2,720 (Fixed Costs) + $90 (Variable Costs) = $2,810 Profit = $3000 - $2,810 = $190 For 4 workers: Quantity of output = 140 units Total Revenue = $25 × 140 = $3500 Total Cost = $960 (Fixed Costs) + $120 (Variable Costs) = $1080 Profit = $3500 - $1080 = $2420 For 5 workers: Quantity of output = 150 units Total Revenue = $25 × 150 = $3750 Total Cost = $2,000 (Fixed Costs) + $160 (Variable Costs) = $2160 Profit = $3750 - $2160 = $1590 For 6 workers: Quantity of output = 155 units Total Revenue = $25 × 155 = $3875 Total Cost = $3,440 (Fixed Costs) + $200 (Variable Costs) = $3640 Profit = $3875 - $3640 = $235 D. The firm should hire 5 workers and can earn a positive profit in the Short Run: This is the correct option.

A Natural Monopoly refers to a monopoly A. market in which there are many sellers. B. market in which there is only one buyer. C. market in which the monopolist faces perfectly elastic demand. D. which arises because one single firm can produce a good at lower total cost than could two or more firms.

D. which arises because one single firm can produce a good at lower total cost than could two or more firms. Explanation: A natural monopoly occurs when a single firm can produce a good or service at a lower total cost than multiple firms. This situation arises due to the presence of significant economies of scale, where the average cost of production decreases as the quantity of output increases. In other words, as the production volume increases, the average cost per unit of the good or service decreases.


संबंधित स्टडी सेट्स

anatomy chapter 6 the integumentary system (SUMMER)

View Set

Chapter 13 Diagnostic Procedures

View Set

ATI Medication Administration Test

View Set

Music Business Ch 17: Concert Venues

View Set

Chapter 6 - Landlord and Tenant (Lessor and Lessee)

View Set